Đến nội dung

whiterose96 nội dung

Có 79 mục bởi whiterose96 (Tìm giới hạn từ 20-04-2020)



Sắp theo                Sắp xếp  

#301153 Giải và biện luận phương trình sau: $$(m-2)x^{2}-2mx+m+1=0$$

Đã gửi bởi whiterose96 on 26-02-2012 - 16:55 trong Phương trình, hệ phương trình và bất phương trình

1.3.Gọi a,b,c là độ dài 3 cạnh của 1 tam giác. Chứng minh phương trình sau vô nghiệm:
$c^{2}x^{^{2}}+(a^{2}-b^{2}-c^{2})x+b^{2}=0$

Bài 1.3:
phương trình có delta(D)= $\left ( a^{2}-b^{2}-c^{2} \right )^{2}-4b^{2}c^{2}$$\left ( a^{2}-b^{2}-c^{2} \right )^{2}-4b^{2}c^{2}$
D=$\left ( a^{2}-b^{2}-c^{2}-2bc \right )\left ( a^{2}-b^{2}-c^{2}+2bc \right )$
D=$\left ( a^{2}-\left ( b+c \right ) ^{2}\right )\left ( a^{2} -\left ( b-c \right )^{2}\right )$
D=$\left ( a-b-c \right )\left ( a+b+c \right )\left ( a-b+c \right )\left ( a+b-c \right )$
Vì a,b,c là độ dài 3 cạnh của 1 tam giác nên D<0 suy ra phương trình vô nghệm



#301155 Chứng minh $\left ( a+b+c \right )^{2}\leq 9bc$

Đã gửi bởi whiterose96 on 26-02-2012 - 17:10 trong Bất đẳng thức và cực trị

Cho a,b,c là độ dài 3 cạnh của một tam giác và $a\leq b\leq c$
CMR: $\left ( a+b+c \right )^{2}\leq 9bc$

___
Công thức kẹp trong cặp dấu $



#301257 Chứng minh $\left ( a+b+c \right )^{2}\leq 9bc$

Đã gửi bởi whiterose96 on 27-02-2012 - 11:01 trong Bất đẳng thức và cực trị

Một lỗi nhỏ ở đây là ta chưa biết $2b-c$ có phải là số dương không

nên không thể suy luận $(2b+c)^2 \leq bc$ tương đương \begin{array}{l}
2b - c \le b\\
2b - c \le c
\end{array}


Ví dụ ta lấy $b=1$ , $c=5$ khi đó \begin{array}{l}
-3 \le 1\\
-3 \le 5
\end{array}


thì $(-3) . (-3) >1 . 5$

theo mình 2b-c là số dương vì $a+b>c
mà a\leq b$ nên 2b-c>0



#301315 $\dfrac{a^2}{(1+a^3)(1+b^3)}+\dfrac{b^2}{(1+b^3)(1+c^3)}+...

Đã gửi bởi whiterose96 on 27-02-2012 - 19:32 trong Bất đẳng thức và cực trị

cho $a, b, c$ là các số dương và abc=8. CMR:

$\dfrac{a^2}{(1+a^3)(1+b^3)}+\dfrac{b^2}{(1+b^3)(1+c^3)}+\dfrac{c^2}{(1+c^3)(1+a^3)} \geq \dfrac{4}{3}$

ta có $4\left ( 1+x^{3} \right )\leq \left ( x^{2}+2 \right )^{2}$ $\Leftrightarrow x^{2}\left ( x-2 \right )^{2}\geq 0$ (luôn đúng)
áp dụng bất đẳng thức trên suy ra
vế trái $\geq \frac{4a^{2}}{\left ( 2+a^{2} \right )\left ( 2+b^{2} \right )}+\frac{4b^{2}}{\left ( 2+b^{2} \right )\left ( 2+c^{2} \right )}+\frac{4c^{2}}{\left ( 2+c^{2} \right )\left ( 2+a^{2} \right )}$
đặt $x=\frac{a^{2}}{4}; y=\frac{b^{2}}{4}; z=\frac{c^{2}}{4}$ khi đó $xyz=1$
ta phải chứng minh $\frac{x}{\left ( 1+2x \right )\left ( 1+2y \right )}+\frac{y}{\left ( 1+2y \right )\left ( 1+2z \right )}+\frac{z}{\left ( 1+2z \right )\left ( 1+2x \right )}\geq \frac{1}{3}$
$\Leftrightarrow x\left ( 1+2z \right )+y\left ( 1+2x \right )+z\left ( 1+2y \right )\geq \frac{1}{3}\left ( 1+2x \right )\left ( 1+2y \right )\left ( 1+2z \right )$
áp dụng bdt AM-GM ta chứng minh được $\left ( 1+2x \right )\left ( 1+2y \right )\left ( 1+2z \right )\geq 3^{3}$
suy ra $2\left ( xy+yz+xz \right )+x+y+z\geq 9$ (đúng do xyz=1)
=> đpcm



#301363 $\frac{ab}{c^{2}a^{2}+c^{2}b^{2}}+\frac{bc}{a^{2}b^{2}+a^{2}c^...

Đã gửi bởi whiterose96 on 27-02-2012 - 22:35 trong Bất đẳng thức và cực trị

Bài 1: Cho 3 số dương a,b,c thỏa mãn abc=1. CMR:
$\frac{ab}{c^{2}a^{2}+c^{2}b^{2}}+\frac{bc}{a^{2}b^{2}+a^{2}c^{2}}+\frac{ac}{b^{2}a^{2}+b^{2}c^{2}}\geq \frac{3}{2}$
Bài 2: Cho a,b,c khác nhau đôi một và ab+bc+ca=4. CMR:
$\frac{1}{\left ( a-b \right )^{2}}+\frac{1}{\left ( b-c \right )^{2}}+\frac{1}{\left ( c-a \right )^{2}}\geq 1$

Công thức kẹp trong cặp dấu $



#301367 $\dfrac{a^2}{(1+a^3)(1+b^3)}+\dfrac{b^2}{(1+b^3)(1+c^3)}+...

Đã gửi bởi whiterose96 on 27-02-2012 - 22:47 trong Bất đẳng thức và cực trị

mình không hiểu cách giải này lắm

áp dụng bất đẳng thức trên phải suy ra
vế trái $\geq \dfrac{16a^2}{(a^2+2)^2(b^2+2)^2}+\dfrac{16b^2}{(b^2+2)^2(c^2+2)^2}+\dfrac{16c^2}{(c^2+2)^2(a^2+2)^2}$
chứ

sr bạn nha mình xem sai đề
mà hình như đề của bạn sai rồi, dưới mẫu có căn ko z? nếu có căn thì làm được theo cách của mình



#301368 $ \frac{a^{2}}{\sqrt{(1+a^{3})(1+b^{3})}}+\frac{b^{2}}{...

Đã gửi bởi whiterose96 on 27-02-2012 - 22:51 trong Bất đẳng thức và cực trị

chứng minh rằng $\frac{a^{2}}{^{\sqrt{(1+a^{3})(1+b^{3})}}}+\frac{b^{2}}{\sqrt{(1+b^{3})(1+c^{3})}}+\frac{c^{2}}{\sqrt{(1+c^{3})(1+b^{3})}}\geq \frac{4}{3}$
với a,b,c là các số dương và abc=8

http://diendantoanhoc.net/index.php?showtopic=68981



#301507 $\dfrac{a^2}{(1+a^3)(1+b^3)}+\dfrac{b^2}{(1+b^3)(1+c^3)}+...

Đã gửi bởi whiterose96 on 28-02-2012 - 22:10 trong Bất đẳng thức và cực trị

uh. mình chép nhầm đề.

nhưng mình không hiểu chỗ

áp dụng AM-GM ta CM được

$(1+2x)(1+2y)(1+2z) \geq 3^3$

suy ra $2\left ( xy+yz+xz \right )+x+y+z\geq 9$ (đúng do xyz=1)
=> đpcm

cái này cm kiểu gì vậy bạn? giải thích giúp mình với!

uhm, đơn giản thôi
áp dụng AM-GM ta có:
$1+2x= 1+x+x \geq 3\sqrt[3]{x^{2}}$
tương tự với 1+2y, 1+2z thì ta có $\left ( 1+2x \right )\left ( 1+2y \right )\left ( 1+2z \right )\geq 3^{3}\sqrt[3]{x^{2}y^{2}z^{2}}=$3^{3}$$ vì xyz=1

còn chứng minh $2\left ( xy+yz+xz \right )+x+y+z\geq 9$ thì ta áp dụng AM-GM cho 9 số xy, yz, xz, xy, yz, xz, x, y, z và có xyz=1 => đpcm



#301516 $\dfrac{a^2}{(1+a^3)(1+b^3)}+\dfrac{b^2}{(1+b^3)(1+c^3)}+...

Đã gửi bởi whiterose96 on 28-02-2012 - 22:46 trong Bất đẳng thức và cực trị

Nhưng như thế VT và VP đều lớn hơn 3 nên đâu so sánh được đâu bạn?

vừa rồi mình viết nhầm đúng phải là $\left ( 1+2x \right )\left ( 1+2y \right )\left ( 1+2z \right )\geq 3^{3}$
dùng tính chất bắc cầu
$x\left ( 1+2z \right )+y\left ( 1+2x \right )+z\left ( 1+2y \right )\geq \frac{1}{3}\left ( 1+2x \right )\left ( 1+2y \right )\left ( 1+2z \right )\geq \frac{1}{3}\times 3^{3}=9$



#301542 $\dfrac{a^2}{(1+a^3)(1+b^3)}+\dfrac{b^2}{(1+b^3)(1+c^3)}+...

Đã gửi bởi whiterose96 on 29-02-2012 - 10:33 trong Bất đẳng thức và cực trị

Hiz Bạn làm gần ra thì lại gặp một lỗi sai, đó là đi sai đường ở đoạn sau.
Phải là
$$\left ( 1+2z \right )+y\left ( 1+2x \right )+z\left ( 1+2y \right )\geq \frac{1}{3}\left ( 1+2x \right )\left ( 1+2y \right )\left ( 1+2z \right ) \Leftrightarrow 2(xy + yz + zx) + x + y + z \ge 8xyz + 1$$ $$ \Leftrightarrow 2(xy + yz + zx) + (x + y + z) \ge 9$$
Cái này chỉ cần áp dụng $AM-GM$ thôi.

mình k hiểu lắm, bạn có thể nói rõ mình sai như thế nào ko?



#301672 $\frac{ab}{c^{2}a^{2}+c^{2}b^{2}}+\frac{bc}{a^{2}b^{2}+a^{2}c^...

Đã gửi bởi whiterose96 on 01-03-2012 - 09:04 trong Bất đẳng thức và cực trị

Hi !!!
Bài 1: Sử dụng đổi biến ;)

Bài 2: Bất đẳng thức cần chứng minh tương đương với:

$\frac{1}{\left ( a-b \right )^{2}}+\frac{1}{\left ( b-c \right )^{2}}+\frac{1}{\left ( c-a \right )^{2}}\geq \frac{4}{ab+bc+ca} $
Ta chú ý rằng: $ \left( {a - c} \right)^2 + \left( {b - c} \right)^2 = \left( {a - b} \right)^2 + 2\left( {a - c} \right)\left( {b - c} \right) $
và $ \left( {a - c} \right)\left( {b - c} \right) \le ab + bc + ca $
Áp dụng bđt AM-GM, ta đc:


$ \sum {\frac{1}{{\left( {a - b} \right)^2 }}} = \frac{1}{{\left( {a - b} \right)^2 }} + \frac{{\left( {a - b} \right)^2 }}{{\left( {a - c} \right)^2 \left( {b - c} \right)^2 }} + \frac{2}{{\left( {a - c} \right)\left( {b - c} \right)}} $

$ \ge \frac{2}{{\left( {a - c} \right)\left( {b - c} \right)}} + \frac{2}{{\left( {a - c} \right)\left( {b - c} \right)}} \ge \frac{4}{{ab + bc + ca}} $

ZZ

bạn có thể nói rõ cách làm bài 1 k?
còn bài 2 chỗ chứng minh $ \left( {a - c} \right)\left( {b - c} \right) \le ab + bc + ca $ mình ko hiểu lắm



#302462 Bạn & Diễn đàn Toán

Đã gửi bởi whiterose96 on 06-03-2012 - 10:54 trong Diễn đàn Toán học trên chặng đường phát triển

hihi, mọi người viết như viết văn ý, dài quá :biggrin:
mình đến vs diễn đàn qua cuốn sáng tạo bất đẳng thức :lol:



#302466 Topic ...............

Đã gửi bởi whiterose96 on 06-03-2012 - 11:02 trong Góc giao lưu

=)) =)) =))



#303051 Tìm min $$\frac{3a}{b+c}+\frac{4b}{c+a}+\frac{5c}{a+...

Đã gửi bởi whiterose96 on 09-03-2012 - 09:26 trong Bất đẳng thức và cực trị

Tìm giá trị nhỏ nhất của biểu thức:
$$\frac{3a}{b+c}+\frac{4b}{c+a}+\frac{5c}{a+b}$$
với a,b,c là các số thực dương
--------------------------------
Công thức toán được kẹp bởi cặp dấu $ bạn nhé.
$cong_thuc$



#303629 Bất Đẳng Thức Qua Các Kỳ TS ĐH

Đã gửi bởi whiterose96 on 11-03-2012 - 19:38 trong Bất đẳng thức và cực trị

Bài 82: Cho 2 số thực dương x,y thay đổi thỏa mãn $3x+y\le1$. Tìm GTNN của biểu thức.
$$A=\frac{1}{x}+\frac{1}{\sqrt{xy}}$$

Đề thi Cao đẳng năm 2010


Áp dụng AM-GM
$\frac{x+y}{2}\geq \sqrt{xy}\Rightarrow \frac{1}{\sqrt{xy}}\geq \frac{2}{x+y}$
$\Rightarrow A\geq \frac{1}{x}+\frac{2}{x+y}\geq 2\sqrt{\frac{2}{x(x+y))}} \Rightarrow A\geq \frac{4}{\sqrt{2x\left ( x+y \right )}}\geq \frac{8}{2x+x+y}\geq 8$
đẳng thức xảy ra khi $x=y=\frac{1}{4}$



#303632 Bất Đẳng Thức Qua Các Kỳ TS ĐH

Đã gửi bởi whiterose96 on 11-03-2012 - 19:45 trong Bất đẳng thức và cực trị

Bài 65: Cho x,y,z là 3 số thực dương. Tìm GTLN của
$P=\dfrac{x}{x+\sqrt{(x+y)(x+z)}}+\dfrac{y}{y+\sqrt{(y+z)(y+x)}}+\dfrac{z}{z+\sqrt{(y+z)(z+x)}}$


áp dụng Cauchy-Schwarz:
${(x+y)(x+z)}\geq \left ( \sqrt{xy} +\sqrt{xz}\right )^{2}$
$\Rightarrow \sqrt{{(x+y)(x+z)}}\geq \left ( \sqrt{xy} +\sqrt{xz}\right )$
$\Rightarrow \frac{x}{x+\sqrt{(x+y)(x+z)}}\leq \frac{\sqrt{x}}{\sqrt{x}+\sqrt{y}+\sqrt{z}}$
Tương tự
$\Rightarrow P\leq 1$
ĐTXR khi x=y=z



#305234 CM $r^{3}+s^{6}\geq 9$

Đã gửi bởi whiterose96 on 19-03-2012 - 11:30 trong Bất đẳng thức và cực trị

Chứng minh rằng nếu r,s >0 và $r^{2}+s^{2}=5 thì r^{3}+s^{6}\geq 9$



#305524 $xyz = 1$ . Chứng minh rằng : $$\dfrac{1}{x^2 + x +...

Đã gửi bởi whiterose96 on 20-03-2012 - 19:59 trong Bất đẳng thức và cực trị

Đặt ẩn phụ kiểu này thôi:
$x=\frac{ab}{c^{2}}$
$y=\frac{bc}{a^{2}}$
$z=\frac{ca}{b^{2}}$
Thay vào sau đó C.S nữa là ổn


Bạn làm chi tiết hơn đi, mình chưa hiểu lắm



#306768 $$\left\{ \begin{array}{l} {x^2} + {y^2} + xy + 1 =...

Đã gửi bởi whiterose96 on 28-03-2012 - 22:31 trong Phương trình - hệ phương trình - bất phương trình

Giải hệ phương trình

\[\left\{ \begin{array}{l}
{x^2} + {y^2} + xy + 1 = 4y\\
y{(x + y)^2} - 2{x^2} - 2 = 7y
\end{array} \right.\]



#306913 $\sum \frac{a^{4}+b^{4}}{ab(a^{3}+b^{3})}\geq 1$

Đã gửi bởi whiterose96 on 29-03-2012 - 22:00 trong Bất đẳng thức và cực trị

Cho a,b,c là các số dương thỏa mãn ab+bc+ca=abc. CMR
$\frac{a^{4}+b^{4}}{ab(a^{3}+b^{3})} + \frac{b^{4}+c^{4}}{bc(b^{3}+c^{3})} + \frac{c^{4}+a^{4}}{ca(c^{3}+a^{3})}\geq 1$


MOD: Công thức toán kẹp trong cặp dấu $



#307806 tìm B, C để chu vi tam giác ABC nhỏ nhất

Đã gửi bởi whiterose96 on 02-04-2012 - 20:23 trong Hình học phẳng

Trong mặt phẳng tọa độ Oxy cho A(1;2) và đường thẳng d: 4x - 3y - 23 = 0. Hai điểm B và C di chuyển trên d sao cho đoạn BC luôn có độ dài bằng 5. Tìm B và C sao cho chu vi tam giác ABC là nhỏ nhất



#308034 Thói quen xấu nên bỏ .

Đã gửi bởi whiterose96 on 03-04-2012 - 21:56 trong Góc giao lưu

anh Trọng viết hay quá :namtay , cảm xúc dâng trào



#309114 Phương trình và hệ phương trình qua các đề thi thử Đại học 2012

Đã gửi bởi whiterose96 on 08-04-2012 - 23:37 trong Phương trình - hệ phương trình - bất phương trình

Bài 8: Giải hệ phương trình: $\begin{cases} {{x}^{2}}+{{y}^{2}}+x+y=4 \\
x(x+y+1)+y(y+1)=2 \end{cases}$

Đề thi thử ĐH trường Hậu Lộc - Thanh Hóa


Hệ đã cho tương đương với:
$$\left\{\begin{matrix} x^{2}+y^{2}+x+y=4\\ x^{2}+y^{2}+x+y+xy=2 \end{matrix}\right.$$
$$\Leftrightarrow \left\{\begin{matrix} (x+y)^{2}+(x+y)-2xy=4\\ xy=-2 \end{matrix}\right.$$
Suy ra
$$\left\{\begin{matrix} x+y=0\\ xy=-2 \end{matrix}\right.$$
hoặc $\left\{\begin{matrix} x+y=-1\\ xy=-2 \end{matrix}\right.$
\[
\Leftrightarrow \left\{ \begin{array}{l}
x = \sqrt 2 \\
y = - \sqrt 2 \\
\end{array} \right. \vee \left\{ \begin{array}{l}
x = - \sqrt 2 \\
y = \sqrt 2 \\
\end{array} \right. \vee \left\{ \begin{array}{l}
x = 1 \\
y = - 2 \\
\end{array} \right. \vee \left\{ \begin{array}{l}
y = - 2 \\
x = 1 \\
\end{array} \right.
\]

Vậy hệ có 4 nghiệm $(x;y)=(\sqrt 2;- \sqrt 2 );(- \sqrt 2 ; \sqrt 2 );(1;-2);(-2;1)$
____
Bạn giải ra tận cùng nhé vì topic này sẽ tổng hợp lại :D



#309132 Phương trình và hệ phương trình qua các đề thi thử Đại học 2012

Đã gửi bởi whiterose96 on 09-04-2012 - 08:05 trong Phương trình - hệ phương trình - bất phương trình

Bài 1:Giải hệ phương trình: $$\begin{cases}x^2 - y(x + y) + 1 = 0\\(x^2 + 1)(x + y - 2) + y = 0 \end{cases}$$
Đề thi thử Đại học THPT Chuyên ĐH Vinh-Lần 2


do $x^{2}+1\neq 0$ nên chia cho $x^{2}+1$ ta được hệ
$\left\{\begin{matrix}
1-\frac{y(x+y)}{x^{2}+1}=0\\
x+y+\frac{y}{x^{2}+1}-2=0
\end{matrix}\right.$
đặt x+y=a; $\frac{y}{x^{2}+1}=b$
$\Rightarrow \left\{\begin{matrix}
a+b=2\\
ab=1
\end{matrix}\right.$
giải hệ trên tìm được x,y
hệ có nghiệm $\left ( 0;1 \right )$ và $\left ( -1;2 \right )$



#311584 Cho A(1;0) và đường trong (C): $x^{2}+y^{2}-2x+4y-5=0$. Xác định...

Đã gửi bởi whiterose96 on 19-04-2012 - 21:50 trong Hình học phẳng

Bài 1: Cho A(1;0) và đường tròn ©: $x^{2}+y^{2}-2x+4y-5=0$. Xác định$(\Delta)$ cắt © tại M, N sao cho tam giác AMN vuông cân tại A

Bài 2: Cho $(\Delta):\sqrt{2}x+my+1-\sqrt{2}=0$ và $©:x^{2}+y^{2}-2x+4y-4=0$
1. Tìm M sao cho $(\Delta)$ cắt © tại 2 điểm phân biệt A,B
2. Gọi I là tâm đường tròn ©. Tính diện tích tam giác IAB (theo m)